Maximum value 1

Algebra Level 4

a 4 + b 4 + c 4 + a b c ( a + b + c ) k ( a b + b c + c a ) 2 \large { a }^{ 4 }+{ b }^{ 4 }+{ c }^{ 4 }+abc\left( a+b+c \right) \ge k{ \left( ab+bc+ca \right) }^{ 2 }

Find the maximum value of k k such that the inequality above holds true with all real values of a , a, b , b, and c . c.

1 3 \frac { 1 }{ 3 } 3 4 \frac { 3 }{ 4 } 2 2 2 3 \frac { 2 }{ 3 }

This section requires Javascript.
You are seeing this because something didn't load right. We suggest you, (a) try refreshing the page, (b) enabling javascript if it is disabled on your browser and, finally, (c) loading the non-javascript version of this page . We're sorry about the hassle.

1 solution

Linkin Duck
Mar 26, 2017

Since the inequality holds true with a = b = c = 1 a=b=c=1 k 2 3 \Longrightarrow k\le \frac { 2 }{ 3 } .

Next, we'll show that k = 2 3 k=\frac { 2 }{ 3 } is achievable, hence 2 3 \frac { 2 }{ 3 } is the maximum value of k k .

When k = 2 3 k=\frac { 2 }{ 3 } , the inequality becomes:

a 4 + b 4 + c 4 + a b c ( a + b + c ) 2 3 ( a b + b c + c a ) 2 ( 1 ) { a }^{ 4 }+{ b }^{ 4 }+{ c }^{ 4 }+abc\left( a+b+c \right) \ge \frac { 2 }{ 3 } { \left( ab+bc+ca \right) }^{ 2 }\quad (1)

3 ( a 4 + b 4 + c 4 ) 2 ( a 2 b 2 + b 2 c 2 + c 2 a 2 ) + a b c ( a + b + c ) { \Longleftrightarrow 3\left( { a }^{ 4 }+{ b }^{ 4 }+{ c }^{ 4 } \right) }\ge 2{ \left( { a }^{ 2 }{ b }^{ 2 }+{ b }^{ 2 }{ c }^{ 2 }+{ c }^{ 2 }{ a }^{ 2 } \right) +abc\left( a+b+c \right) }

By applying AM-GM inequality,

( a 4 + b 4 ) + ( b 4 + c 4 ) + ( c 4 + a 4 ) 2 a 2 b 2 + 2 b 2 c 2 + 2 c 2 a 2 3 ( a 4 + b 4 + c 4 ) 3 ( a 2 b 2 + b 2 c 2 + c 2 a 2 ) ( 2 ) \left( { a }^{ 4 }+{ b }^{ 4 } \right) +\left( { b }^{ 4 }+{ c }^{ 4 } \right) +\left( { c }^{ 4 }+{ a }^{ 4 } \right) \ge 2{ a }^{ 2 }{ b }^{ 2 }+2{ b }^{ 2 }{ c }^{ 2 }+2{ c }^{ 2 }{ a }^{ 2 }\\ \Longrightarrow 3\left( { a }^{ 4 }+{ b }^{ 4 }+{ c }^{ 4 } \right) \ge 3\left( { a }^{ 2 }{ b }^{ 2 }+{ b }^{ 2 }{ c }^{ 2 }+{ c }^{ 2 }{ a }^{ 2 } \right) \quad (2)

On the other hand,

a 2 b 2 + b 2 c 2 + c 2 a 2 a b c ( a + b + c ) = 1 2 ( a b b c ) 2 + 1 2 ( b c c a ) 2 + 1 2 ( c a a b ) 2 0 ( 3 ) { a }^{ 2 }{ b }^{ 2 }+{ b }^{ 2 }{ c }^{ 2 }+{ c }^{ 2 }{ a }^{ 2 }-abc\left( a+b+c \right) =\frac { 1 }{ 2 } { \left( ab-bc \right) }^{ 2 }+\frac { 1 }{ 2 } { \left( bc-ca \right) }^{ 2 }+\frac { 1 }{ 2 } { \left( ca-ab \right) }^{ 2 }\ge 0\quad (3)

From ( 2 ) (2) and ( 3 ) (3) , ( 1 ) (1) is proved.

So k m a x = 2 3 . { k }_{ max }=\boxed { \frac { 2 }{ 3 } } .

Very nice problem!

James Wilson - 3 years, 9 months ago

@Linkin Duck - To prove (2) and (3) could you also use the rearrangement inequality?? Thanks!

Christopher Criscitiello - 3 years, 8 months ago

I tried to solve this by considering a cubic which has roots a,b and c. But failed badly. Then I did exactly this thing. Is it possible to solve some difficult inequalities by considering some nth degree polynomial.

Srikanth Tupurani - 2 years, 7 months ago

0 pending reports

×

Problem Loading...

Note Loading...

Set Loading...